48 - (3 c+4)=4(c+5)+3 c=???

Answers

Answer 1

Answer in multiple forms depending on what needed

48 - (3 C+4)=4(c+5)+3 C=???
Answer 2

Answer:

c = 3

Step-by-step explanation:

48- (3c+4)=4(c+5)+3

48-3c-4=4(c+5)+3 Distributed the - sign

48 -3c -4= 4c+20 +3 Distributed the 4

44-3c =4c+23 Combined like terms

21-3c=4c Subtract 23 from both sides

21=7c Add 3c to both sides

3=c


Related Questions

Find the sum: (2x² + x + 3) + (3x² + 2x + 1)
A. 3x² + 5x+4
B. 5x²+3x+4
OC. 6x + 2x + 3
OD. 5x²+2x+4

Answers

Answer:

B

Step-by-step explanation:

(2x²+x+3) + (3x²+2x+1)

Combine 2x^2 and 3x^2 to get 5x^2.

5x²+x+3+2x+1

Combine x and 2x to get 3x.

5x²+3x+3+1

Add 3 and 1 to get 4.

5x²+3x+4The correct option is B.

Find P(AUB) of P(A)=2/5,P(B)=2/7 and A and B are Independent

Answers

P(A∪B)=24/35 when A and B are independent and P(A)=2/5,P(B)=2/7

What is meant by probability?

Probability is the branch of mathematics that deals with numerical descriptions of how likely an event is to occur or how likely it is that a claim is true. The probability of an event is a number between 0 and 1, with 0 signifying impossibility and 1 expressing certainty. The greater the likelihood of an occurrence, the more likely it will occur.

Probability theory, which is widely used in domains such as statistics, mathematics, science, finance, gambling, artificial intelligence, machine learning, computer science, game theory, and philosophy, has given these concepts axiomatic mathematical formalization.

Given,

P(A)=2/5

P(A)=2/7

P(A∪B)=?

And also given that A and B are independent then,

P(A∩B)=0

P(A∪B)=P(A)+P(B)-P(A∩B)

P(A∪B)=(2/5)+(2/7)-0

P(A∪B)=(14+10)/35

P(A∪B)=24/35

Therefore, P(A∪B)=24/35 when A and B are independent.

To know more about probability, visit:

https://brainly.com/question/11234923

#SPJ1

There are 3 trees in your neighbor's yard. Each tree in the yard has 2 birdhouses and each birdhouse has 4 birds. How many birds are
there in all?

Answers

There are 24 birds in all
Explanation: you multiply 3x2 which is 6 then you multiply 4 so 6x4 = 24.

Differentiate the function with respect to x.

Answers

Differentiation of f(x) is [tex]-8x^{3} sinx^{4}[/tex].

Given,

f(x) = [tex]cos2x^{4}[/tex]

Now, we will differentiate f(x) with respect to x,

[tex]\frac{d}{dx}cos2x^{4} = (\frac{d}{dx}cos2x^{4})(\frac{d}{dx}2x^{4} )[/tex]

              [tex]= (-sin2x^{4})(2*4x^{3})[/tex]

              [tex]= -8x^{3}sin2x^{4}[/tex]

To learn more about differentiation here:

https://brainly.com/question/24898810

#SPJ1

Hi can someone please help me? Thank you!

Answers

103mm is the correct answer

Can anyone help me in this question? pls.........

Answers

Answer: A

Step-by-step explanation :

HE ASKED YOU ABOUT AN EQUALTULAR ( TO EQUALED AND OPPSITE ANGLE )

Question 5 (1 point), You are making your friend a triangular fabric decoration for their home. It has ripped so you need to go buy new fabric. The fabric you are buying is $3 per square foot. Using the dimensions below, how much money do you need to buy enough fabric? ​

Answers

Answer:

1). True

2). x = 16.4 m^2

3). A = 24, h = 4

4). A = 18.39

5). 162

Step-by-step explanation:

I took this quiz on K12 and got everything correct! Brainliest welcome! :)

Describe a real life siuation for each problem
1. -3 + (-4)
2. -3 (+2)

Answers

Answer:

1. I'm $3 in debt, and I spend 4 more dollars. I am now 4 dollars more in debt than I was before.

2. I'm $3 in debt, and I pay back $2 of debt. I am now $1 in debt.

Step-by-step explanation:

-Hope this helped

Can someone please help me , thank you :)!

Answers

Answer:

a )

[tex]6 ( ? ) = 27[/tex]

b )

? = The amount of rice in each container.

So, 1 container = ( ? ) ounce.

6 containers have = 6 x ( ? ) ounces.

We know that 6 containers have 27 ounces so...

[tex]6 ( ? ) = 27[/tex]

[tex]? = \frac{27}{6}[/tex]

[tex]? = 4.5[/tex]

Final Answer: 4.5 ounces in each container

Hope this helps! :)

In 2016 the cdc estimated the mean weight of us women over the age of 20 years was 168.5 pounds with a standard deviation of 68 pounds. What is the expected mean for a sample of 150 women

Answers

The expected mean for a sample of 150 women is 168.5 pounds.

What is expected mean?

Expected mean can be defined as the mean weight of a number . Based on the given data we were told that the mean weight of the us women was 168.5 pounds which implies that the expected mean is 168.5 pounds.

The formula for expected mean is :

E ( X ) = μ = ∑ x P ( x )

Where:

x = values of the random variable X

P(x) = Corresponding probability

∑ =Sum of all products xP(x)

μ = mean

Therefore the expected mean is 168.5 pounds.

Learn more about expected mean here: https://brainly.com/question/10675141

#SPJ1

Which describes the graph of y=-(x - 3)² + 2?
O A. Vertex at (-3,2)
OB. Vertex at (3,-2)
OC. Vertex at (-3, -2)
OD. Vertex at (3, 2)

Answers

x-3=0

so x = 3

-(y-2)=0

so y = 2

vertex = (3,2)

What is 4/5 ÷ 1/3? The quotient is 2 and​

Answers

Answer:4/5 : 1/3=4/5 x 3/1=12/5

Step-by-step explanation:

Answer:

12/5

Step-by-step explanation:

If you set it in the calculator 4/5 ÷ 1/3 you get = 12/5

was it helpful?

Please help with this table. Not sure why this is difficult ​

Answers


in order: -4, 0.75, 1.25, -13

if x represents the number of questions on test, analyze the meaning of each expression x+4

Answers

The meaning of the expression x + 4 as required in the task content is; "four more than the number of questions on test".

Determining word phrases from algebraic expressions.

It follows from the task content that the word phrase which best represent the algebraic expression; x + 4 is to be determined.

Given; x = the number of questions on test.

It therefore follows that the algebraic expression x + 4 can be interpreted as; "four more than the number of questions on test".

Hence, the meaning of the expression is; "four more than the number of questions on test".

PS: Just like word phrases can be accurately represented by algebraic expressions, the latter can be represented by the former too.

Read more on algebraic expressions from word phrases;

https://brainly.com/question/28345080

#SPJ1

Translate the sentence into an equation.
The square of a number is equal to the square root of that number.

Answers

The equation of the sentence, the square of a number is equal to the square root of that number, is [tex]x^{2} =\sqrt{x}[/tex].

According to the question,

We have the following information:

The square of a number is equal to the square root of that number.

Now, let's take this number to be x.

Now, square of this number will be [tex]x^{2}[/tex] and the square root of this number will be [tex]\sqrt{x}[/tex].

Now, according to the sentence, we will have the following expression:

[tex]x^{2} =\sqrt{x}[/tex]

Hence, the equation of the sentence, the square of a number is equal to the square root of that number, is [tex]x^{2} =\sqrt{x}[/tex].

To know more about equation here

https://brainly.com/question/23384508

#SPJ1

A car dealership decreased the price of a certain car by 5%. The original price was $47,200.
What is the new value and the original value?

Answers

Answer:

New: $44,840 Original: 47,200

Step-by-step explanation:

You have to multiply 47,200 by 95% or 0.95

If A and B are independent events, P(A)=0.15, and P(B)=0.77, what is P(B|A)?

Answers

The Probability of P(B│A) is 0.77

What are independent events?

Independent events are those events whose occurrence is independent on any other event. For example, if we flip a coin in the air and get the outcome as tail, then again if we flip the coin but this time we get the outcome as Head, In both cases, the occurrence of both events is independent of each other. It is a type of events in probability.

If A is the event that even number will appear when a die is rolled and B is when 4 or 6 appears

P(A)= 3/6 = 1/2 and P(B) = 2/6 = 1/3

Also A and B is the event ‘the number appearing is even and a multiple of 4 or 6 so that

P(A ∩ B) = 1/6

P(B│A) = P(A ∩ B)/ P(A)

= 1/6 /1/2= 1/3 = P(B)

Therefore P(B│A) = P(B)

if A= 0.15 and P(B)= 0.77

P(B│A) = 0.77

learn more about independent event from

https://brainly.com/question/14106549

#SPJ1

Write the equation of the line that passes through the points (5,—9) and (-7, 7).
Put your answer in fully simplified point-slope form, unless it is a vertical or
horizontal line.

Answers

Answer:

[tex]y-7=-\frac{4}{3}(x+7)[/tex]

Step-by-step explanation:

The slope of the line is [tex]\frac{-9-7}{5-(-7)}=-\frac{4}{3}[/tex].

The equation of the line passing through the point [tex](x_1, y_1)[/tex] and slope [tex]m[/tex] is [tex]y-y_1=m(x-x_1)[/tex], so the equation is [tex]y-7=-\frac{4}{3}(x+7)[/tex].

it costs a paper company 4x+185 dollars to produce x notebooks. how many notebooks can they produce for $285

Answers

The paper company can produce 25 notebooks for $285

How to determine the number of notebooks?

From the question, the equation of the total cost is given as

Equation = 4x + 185

Rewrite as

Cost = 4x + 185

Express the equation as a cost function

So, we have the following representations

C(x) = 4x + 185

When the total cost is $285, we have

C(x) = 285

This gives

4x + 185 = 285

Subtract 185 from both sides

4x = 100

Divide by 4

x  = 25

Hence, the number of notebooks is 25

Read more about linear equations at

https://brainly.com/question/2030026

#SPJ1

Help Geometry (ASAP!!)​

Answers

Answer:

yes if b is odd then b+6 is odd

Step-by-step explanation:

example

consider a odd number 1,3,5,7,and 9

1+6=7

3+6=9

5+6=11

7+6=13

9+6=15

all answers are odd number


589 in base five is

Answers

589 in base five is  [tex]4324_{5}[/tex].

Given number:

589

converting to [tex]589_{5}[/tex].

⇒ 5|589

⇒ 5|117|4

⇒ 5|23|2

⇒ 5|4|3

⇒ 5|4|4

Now we write the last digit from last number to first that is,

= [tex]4324_{5}[/tex].

Hence 589 in base five is  [tex]4324_{5}[/tex].

Learn more about the base five here:

https://brainly.com/question/28645823

#SPJ1

The marginal cost (dollars) of printing a poster when x posters have been printed is given by the following equation.
-3/4
C'(x)=x
Find the cost of printing 133 more posters when 17 have already been printed.
The cost of printing 133 more posters when 17 have already been printed is $.
(Round to the nearest cent as needed.)

Answers

When only 17 posters have been printed, it will cost $ 5. 46to print 143 more.

How to calculate cost of painting?

A mathematical formula known as a cost function is used to visualise how manufacturing costs will vary depending on the output level. In other words, given a specified quantity produced, it calculates the entire cost of production.

The following equation, C'(x)=x-3/4, tells us the marginal cost (in dollars) of printing a poster after x posters have been printed.

The given equation is :C'(x)=x-3/4

The cost of printing 133 more posters when 17 have already been printed is given by;

applying the limitations we obtain and integrating both sides of the equation;

[tex]$$\int_a^b C^{\prime}(x) d x=\int_{17}^{133} x^{\frac{-3}{4}} d x$$[/tex]

As we know that [tex]$\int x^n d x=\frac{x^{n+1}}{n+1}$[/tex], so;

[tex]$&=4\left[x^{\frac{1}{4}}\right]_{17}^{133} \\[/tex]

[tex]$&=4\left[(133)^{\frac{1}{4}}-(17)^{\frac{1}{4}}\right] \\[/tex]

= $ 5.4616

Therefore When only 17 posters have been printed, it will cost $ 5. 46to print 143 more.

To learn more about  cost refer to:

https://brainly.com/question/2292799

#SPJ1

An auto insurance company charges males 16-18 years old a base price of $1100 for insurance in a given year. Their accuracies have determined that there is a 8.5% a claim will be made against such a policy and an average claim of $4900. What does the insurance company expect to make on average off of each policy holder?

Answers

If a claim will be made against such a policy and an average claim of $4900. the insurance company should make on average off of each policy holder is $683.5.

How to find the amount the company should make?

Using this formula to find the amount

Amount = [Percentage claim × ( Base price - Average claim)] + [(1 - percentage claim) × Base price]

Let plug in the formula

Amount = [0.085 × ($1,100 - $4,900)] + [(1-0.085) × $1,100]

Amount = [0.085 ×  (- $3,800)] + [(0.915 × $1,100]

Amount = -$325+ $1,006.5

Amount = $683.5

Therefore the insurance company will make the amount of $683.5.

Learn more about insurance here:https://brainly.com/question/25855858

#SPJ1

Quinn is baking sweet potato pies. The table shows the ratio of cups of sugar to number of pies.


Number of Pies 3 5 9
Cups of Sugar 1 and one half 2 and one half 4 and one half


How many cups of sugar will Quinn need to make 12 pies?
5 and one half cups
6 cups
6 and one half cups
8 cups

Answers

The ratio of 6 cups of sugar to pie that Quinn needs to make 12 potato pies will be 6 cups.

What is ratio?

Ratio basically compares quantities, that shows value of one quantity with respect to other quantity.

If a and b are two values, there ratio will be a:b.

Given that,

For 3 potato pies required sugar is 3/2,

For 5 potato pies required sugar is 5/2,

for 9 potato pies required sugar is 9/2,

From the given information,

For 1 potato pies required sugar is 1/2

⇒For 12 potato pies required sugar=12×1/2=6

To make 12 potato pies,  required sugar will be 6.

To learn more on Ratio :

https://brainly.com/question/13419413

#SPJ1

A rotating light is located 17 feet from a wall. The light completes one rotation every 2 seconds. Find the rate at which the light projected onto the wall is moving along the wall when the light's angle is 20 degrees from perpendicular to the wall.

Answers

When the the light's angle is 20° from perpendicular to the wall, and the light is rotating at an angular speed of one rotation in 2 seconds the rate the light projected onto the wall is moving is approximately 60.48 feet/s

What is angular speed?

Angular speed, ω, is the angle, θ, in radians rotated by a rotating object per second. The unit of angular speed is radians per second, rad/s.

Mathematically;

[tex]\omega = \dfrac{\Delta \theta}{\Delta t}[/tex]

Where;

Δt = The time the object rotates through an angle of Δθ

The distance of the light from the wall, x = 17 feet

Time it takes the light to complete one rotation = 2 seconds

Angle the light makes with the perpendicular wall = 20°

[tex]tan(\theta) = \dfrac{y}{x}[/tex]

The distance the light has moved along the wall, y = x × tan(θ)

Therefore;

The y = 17 × tan(θ)

The angular velocity of the light, ω = θ/t

Therefore, θ = ω × t = ω·t

Which gives; y = 17 × tan(ω·t)

[tex]\omega = \dfrac{2\cdot \pi}{2}\ rad/s = \pi \ rad/s[/tex]

Therefore; x = 17 × tan(·t)

[tex]\dfrac{dy}{dt} = \dfrac{d}{dt} \left(17\times tan(\omega\cdot t)\right) = 17\cdot \omega\cdot sec^2(\omega \cdot t)[/tex]

ω = π rad/s (constant)

ω·t = θ

Therefore;

[tex]\dfrac{dy}{dt}= 17\cdot \omega\cdot sec^2(\theta)[/tex]

Rate the light is moving along the wall, dy/dt when θ = 20°, is therefore;

[tex]\dfrac{dy}{dt}= 17\times \pi\cdot sec^2(20^{\circ}) \approx 60.48[/tex]

The light is moving at a rate of 60.48 feet/s when the lights angle is 20 degrees from the perpendicular wall

Learn more about angular or rotational speed here:

https://brainly.com/question/13014974

#SPJ1

Pedro and Rebecca were trying to solve the equation:

0.1x^2 - 0.7x + 1.2 = 0
Pedro said, “I can multiply the entire equation by 10 and then factor. Then I can solve using the zero product property.”

Rebecca said, “I'll solve using the quadratic formula with a = 0.1 1, b = -0.7 and c = 1.2"

WHOs Strategy works?
A - Only Pedro's

B - Only Rebecca's

C - Both

D - Neither

Answers

The correct strategy between Pedro and Rebecca trying to solve the quadratic equation: 0.1x^2 - 0.7x + 1.2 = 0 is A - Only Pedro's

How to determine the correct strategy

The given function is a quadratic function which is solved by several method including the method both Pedro and Rebecca are trying to use

Pedro's method is the factorization method and the steps mentioned are correct

However Rebecca's method has some error

The equation is: 0.1x^2 - 0.7x + 1.2 = 0

factor of the leading coefficient a is 0.1, Rebecca is using 0.11

The error of using 0.11 instead of 0.1 will make the method ineffective

Learn more about quadratic equations here:

https://brainly.com/question/29116024

#SPJ1

What is the value of x?

Answers

The required value of x is 3 in the given equilateral triangle.

What is a Triangle?

Triangle is defined as a basic polygonal shape of a triangle that has three sides and three interior angles.

The triangle given in the figure is an equilateral triangle.

We know that all sides are equal in the equilateral triangle.

As per the given figure,

side AB = side AC

6x - 3 = 3x + 6

6x - 3x = 6 + 3

Apply the arithmetic operation,

3x = 9

x = 9/3

x = 3

Therefore, the required value of x is 3 in the given equilateral triangle.

Learn more about the triangles here:

https://brainly.com/question/17997149

#SPJ1

earns $14.50 per hour. His regular hours
are 40 hours per week, and he receives
time-and-a-half overtime. Find his total pay
for a week in which he works 46 hours.

Answers

Answer: 667

Step-by-step explanation:

14.50 x 46

In a photo contest, two judges are giving scores to each photo. Each judge gives a score for impact (worth 40%), composition (worth 30%), lighting (worth 20%), and color balance (worth 10%). Lester entered a photo in the contest and received the following scores: Impact: 8 and 9 Composition: 7 and 7 Lighting: 8 and 7 Color balance: 5 and 7 Enter a number in each box to create an expression that represents Lester's final score.

Answers

Lester's final score from the two Judges is 7.6

How to calculate Lester's final score

The scores of the judges are in percentage and this will be converted to decimal by dividing by 100

impact (worth 40%) = 0.4

composition (worth 30%) = 0.3

lighting (worth 20%) = 0.2

color balance (worth 10%) = 0.1

Average score from the Judges will be used to multiply the percentages

Impact: 8 and 9

= 8.5 * 0.4 = 3.4

Composition: 7 and 7

= 7 * 0.3 = 2.1

Lighting: 8 and 7

= 7.5 * 0.2 = 1.5

Color balance: 5 and 7

= 6 * 0.1 = 0.6

sum of the scores gives Lester's final score as 7.6

Learn more about percent here:

https://brainly.com/question/29230566

#SPJ1

. You get an auto loan at 3.5% interest for 5 years; please answer the following.
A. You can afford a $250 per month car payment; how expensive a car can you afford?
B. You purchase a car for $8000 with a $2000 down payment, what are your monthly payments?

Answers

Using the monthly payment formula, it is found that:

A. You can afford a car at a price of $13,742.36.

B. The monthly payments are of $109.15.

What is the monthly payment formula?

The monthly payments are given by the equation presented as follows:

[tex]A = P\frac{\frac{r}{12}\left(1 + \frac{r}{12}\right)^n}{\left(1 + \frac{r}{12}\right)^n - 1}[/tex]

The variables involved in the equation are listed and explained as follows:

P is the initial amount.r is the interest rate.n is the number of payments.

For item a, the parameters are given as follows:

A = 250, r/12 = 0.035/12 = 0.002917, n = 5 x 12 = 60

Hence the maximum value of the car is obtained as follows:

[tex]A = P\frac{\frac{r}{12}\left(1 + \frac{r}{12}\right)^n}{\left(1 + \frac{r}{12}\right)^n - 1}[/tex]

[tex]250 = P\frac{0.002917(1.002917)^{60}}{(1.002917)^{60} - 1}[/tex]

0.0181919241P = 250

P = 250/0.0181919241

P = $13,742.36

For item b, the parameters are given as follows:

P = 6000, r/12 = 0.035/12 = 0.002917, n = 5 x 12 = 60

(P = 6000 as the 2000 of the down payment are already paid, hence not subjected to interest).

Hence the monthly payments are given as follows:

[tex]A = P\frac{\frac{r}{12}\left(1 + \frac{r}{12}\right)^n}{\left(1 + \frac{r}{12}\right)^n - 1}[/tex]

[tex]A = 6000\frac{0.002917(1.002917)^{60}}{(1.002917)^{60} - 1}[/tex]

A = 6000 x 0.0181919241

A = $109.15.

More can be learned about the monthly payment formula at https://brainly.com/question/26011426

#SPJ1

Other Questions
11. 1156 12. 1024 13. 1225 14. 1764 15. 1936please ahh little help please Suzette ran and biked for a total of 50 mi in 4 h. Her average running speed was 5mph and her average biking speed was 15 mph. Let x= total hours ran Let y= total hours biked How many hours did suzette run? How many hours did she bike? 10. In which of the following situations is a biotechnologist most likely to decide to use directed evolution as an enzyme engineering technique as opposed to rational design?A) When a structure of theenzyme is available to aid engineeringB) When the enzyme can only bescreened in a low throughput mannerC) When the enzyme mechanismis understood and provides insight to engineeringD) When current data allowsmolecular modelling techniques to be used predictivelyE)When none of the above are true The evolution of Drug-Resistance HIV. Within a few weeks of treatment with drug X, a patient's HIV population consists entirely of X-resistant HIV. Explain how this rapid evolution of drug resistance is an example of natural selection. what is a person separate from a group? how and when do we establish an identity outside of others? what makes you who you are? cite evidence from this text, your own experience, and other literature, art, or history in your answer. Can someone please help with my college music homework? Two possible causes of temperature changes between 400,000 and 100,000 years ago Which sentence is punctuated correctly? a. In the first place modern communicators have made it possible for many people to work from home. b. In the first place modern communicators have made it possible, for many people, to work from home. c. In the first place modern communicators, have made it possible for many people to work from home. d. In the first place, modern communicators have made it possible for many people to work from home. e. There __ good books out there. what are project deliverables? multiple choice a temporary endeavor undertaken to create a unique product or service the application of knowledge, skills, tools, and techniques to project activities in order to meet or exceed stakeholder needs and expectations from a project any measurable, tangible, verifiable outcome, result, or item that is produced to complete a project or part of a project key dates when a certain group of activities must be performed there are 46 students going on a field trip to the museum. if 7 students can fit in each van , how many vans are needed for all 46 students to go to the museum a customer requests a $5,000 partial surrender of his universal life insurance policy. the agency producer notices that the customer has made a $5,000 premium deposit just a week before. the producer should: The process of learning academic facts and concepts through curriculum is called ____ education, and the process of learning about cultural values, norms and expected behaviors is called _____ education. pls i will give top rating 22. A train travels 3 hours at 20km perhour. How long will it take anothertrain to travel the same distance at10km per hour?(a) 6/ hours(b) 6/2hours(c) 6 hours(d) 3 hours(e) 2 hoursWhat I PLS HURRY I NEED THE ANSWER FAST!!!!!!Paula makes purses. The materials cost $2.25 per purse. She sells them for $5.00 each. Paula wants to invest $5000 for some new equipment. Which of these could be used to determine x, the number of purses Paula must sell to pay for the new equipment?A 5.00x - 2.25x = 5000B 5.00x + 2.25x = 5000C 5.00x = 5000D 2.25x = 5000 WILL GIVE BROANLYEST 80 POINTS!!!!!!! What is the volume of a rectangular prism with a length of twenty-two and one-half feet, a width of 12 feet, and a height of 13 feet? 7,800 ft3 3,510 ft3 1,755 ft3 five hundred sixty-two and one-half ft3 You have a gift certificate to a book store worth $95. Each paperback books is $10 and each hardcover books is $17. You must spend at least $20 in order to use the gift certificate. Write and graph a system of inequalities to model the number of each kind of books you can buy. Let x = the number of paperback books and y = the number of hardback books Which action represents a break with George Washington's policies? 1. A cone-shaped filter has a diameter of 10 inches and a height of 4 inches.What is the volume of liquid the filter can hold? Round your answer tothe nearest hundredth. Please Help Me Fast This Is Due Tomorrow!!!!!!!!! Arjun bought a trading card for $12. Five years later, the card is now worth $25. By what percent did the worth of the card increase? Round to the nearest whole percent, if needed.